0
$\begingroup$

Suppose $R$ is a regular local ring and $I$ is a non-zero ideal such that $I$ is a radical ideal and $I$ is height unmixed. Suppose $J$ is any radical ideal contained in $I$ and with the same height as $I$. Can we always find a prime $P\in \operatorname{Ass}(R/J)$ such that $P$ is not minimal.

$\endgroup$
0

1 Answer 1

0
$\begingroup$

$I=(x) \cap (y)$, $J = (x) \cap (y) \cap (z)$.

$\endgroup$
0

Your Answer

By clicking “Post Your Answer”, you agree to our terms of service and acknowledge you have read our privacy policy.

Not the answer you're looking for? Browse other questions tagged or ask your own question.